Outils pour utilisateurs

Outils du site


math:2:demo:serie_riemann

Preuve : séries de Riemann

  • $\boxed{\alpha\leqslant0}$ : La suite $\ds\left(\frac{1}{n^{\alpha}}\right)_{n\geqslant1}$ ne converge pas vers 0 donc la série est divergente.
    Par ailleurs, la fonction $\ds t\mapsto\frac{1}{t^{\alpha}}=t^{-\alpha}$ est croissante sur $]0,+\infty[$ donc :
    $$\ds\forall k\in\N^{*},\;\forall t\in[k,k+1],\;\frac{1}{k^{\alpha}}\leqslant\frac{1}{t^{\alpha}}\leqslant\frac{1}{(k+1)^{\alpha}}$$Le théorème IAF donne alors :
    $$\ds\forall k\in\N^{*},\;\frac{1}{k^{\alpha}}\leqslant\frac{(k+1)^{1-\alpha}}{1-\alpha}-\frac{k^{1-\alpha}}{1-\alpha}\leqslant\frac{1}{(k+1)^{\alpha}}$$Pour tout entier $n\geqslant1$, on somme ces inégalités du rang 1 au rang $n$ et, par télescopage, on obtient :
    $$\ds\sum_{k=1}^{n}{\frac{1}{k^{\alpha}}}\leqslant\frac{(n+1)^{1-\alpha}}{1-\alpha}-\frac{1}{1-\alpha}\leqslant\sum_{k=1}^{n}{\frac{1}{(k+1)^{\alpha}}}$$$$\ds\sum_{k=1}^{n}{\frac{1}{k^{\alpha}}}\leqslant\frac{(n+1)^{1-\alpha}}{1-\alpha}-\frac{1}{1-\alpha}\leqslant\sum_{k=2}^{n+1}{\frac{1}{k^{\alpha}}}$$Pour tout $n\geqslant2$, on a donc :\\ $$\ds\frac{n^{1-\alpha}}{1-\alpha}-\frac{\alpha}{1-\alpha}\leqslant\sum_{k=1}^{n}{\frac{1}{k^{\alpha}}}\leqslant\frac{(n+1)^{1-\alpha}}{1-\alpha}-\frac{1}{1-\alpha}$$La première inégalité rappelle que la série diverge puisque $\ds\frac{n^{1-\alpha}}{1-\alpha}-\frac{\alpha}{1-\alpha}\xrightarrow[n\to+\infty]{}+\infty$ (car $1-\alpha>0$). De plus :\\ $$\ds1-\frac{\alpha}{n^{1-\alpha}}\leqslant\frac{\ds\sum_{k=1}^{n}{\frac{1}{k^{\alpha}}}}{\ds\frac{n^{1-\alpha}}{1-\alpha}}\leqslant\left(1+\frac{1}{n}\right)^{1-\alpha}-\frac{1}{n^{1-\alpha}}$$À gauche et à droite, on a la convergence vers 1, d'où l'équivalent de la somme partielle de rang $n$ de la série.
  • $\boxed{0<\alpha<1}$ : La fonction $\ds t\mapsto\frac{1}{t^{\alpha}}=t^{-\alpha}$ est décroissante sur $]0,+\infty[$ donc :
    $$\ds\forall k\in\N^{*},\;\forall t\in[k,k+1],\;\frac{1}{(k+1)^{\alpha}}\leqslant\frac{1}{t^{\alpha}}\leqslant\frac{1}{k^{\alpha}}$$Le théorème IAF donne alors :
    $$\ds\forall k\in\N^{*},\;\frac{1}{(k+1)^{\alpha}}\leqslant\frac{(k+1)^{1-\alpha}}{1-\alpha}-\frac{k^{1-\alpha}}{1-\alpha}\leqslant\frac{1}{k^{\alpha}}$$Pour tout entier $n\geqslant1$, on somme ces inégalités du rang 1 au rang $n$ et, par télescopage, on obtient :
    $$\ds\sum_{k=1}^{n}{\frac{1}{(k+1)^{\alpha}}}\leqslant\frac{(n+1)^{1-\alpha}}{1-\alpha}-\frac{1}{1-\alpha}\leqslant\sum_{k=1}^{n}{\frac{1}{k^{\alpha}}}$$$$\ds\sum_{k=2}^{n+1}{\frac{1}{k^{\alpha}}}\leqslant\frac{(n+1)^{1-\alpha}}{1-\alpha}-\frac{1}{1-\alpha}\leqslant\sum_{k=1}^{n}{\frac{1}{k^{\alpha}}}$$Pour tout $n\geqslant2$, on a donc :\\ $$\ds\frac{(n+1)^{1-\alpha}}{1-\alpha}-\frac{1}{1-\alpha}\leqslant\sum_{k=1}^{n}{\frac{1}{k^{\alpha}}}\leqslant\frac{n^{1-\alpha}}{1-\alpha}-\frac{\alpha}{1-\alpha}$$La première inégalité prouve que la série diverge puisque $\ds\frac{(n+1)^{1-\alpha}}{1-\alpha}-\frac{1}{1-\alpha}\xrightarrow[n\to+\infty]{}+\infty$ (car $1-\alpha>0$). De plus :\\ $$\ds\left(1+\frac{1}{n}\right)^{1-\alpha}-\frac{1}{n^{1-\alpha}}\leqslant\frac{\ds\sum_{k=1}^{n}{\frac{1}{k^{\alpha}}}}{\ds\frac{n^{1-\alpha}}{1-\alpha}}\leqslant1-\frac{\alpha}{n^{1-\alpha}}$$À gauche et à droite, on a la convergence vers 1, d'où l'équivalent de la somme partielle de rang $n$ de la série.
  • $\boxed{\alpha=1}$ : La fonction $\ds t\mapsto\frac{1}{t}$ est strictement décroissante sur $]0,+\infty[$ donc :
    $$\ds\forall k\in\N^{*},\;\forall t\in[k,k+1],\;\frac{1}{k+1}\leqslant\frac{1}{t}\leqslant\frac{1}{k}$$Le théorème IAF donne alors :
    $$\ds\forall k\in\N^{*},\;\frac{1}{k+1}\leqslant\ln(k+1)-\ln(k)\leqslant\frac{1}{k}$$Pour tout entier $n\geqslant1$, on somme ces inégalités du rang 1 au rang $n$ et, par télescopage, on obtient :
    $$\ds\sum_{k=1}^{n}{\frac{1}{k+1}}\leqslant\ln(n+1)-\ln(1)\leqslant\sum_{k=1}^{n}{\frac{1}{k}}$$$$\ds\sum_{k=2}^{n+1}{\frac{1}{k}}\leqslant\ln(n+1)\leqslant\sum_{k=1}^{n}{\frac{1}{k}}$$Pour tout $n\geqslant2$, on a donc :\\ $$\ds\ln(n+1)\leqslant\sum_{k=1}^{n}{\frac{1}{k}}\leqslant\ln(n)+1$$La première inégalité assure la divergence de la série puisque $\ln(n+1)\xrightarrow[n\to+\infty]+\infty$. De plus :\\ $$\ds1+\frac{\ln\left(1+\frac{1}{n}\right)}{\ln(n)}\leqslant\frac{\ds\sum_{k=1}^{n}{\frac{1}{k}}}{\ln(n)}\leqslant1+\frac{1}{\ln(n)}$$À gauche et à droite, on a la convergence vers 1, d'où l'équivalent de la somme partielle de rang $n$ de la série.
  • $\boxed{\alpha>1}$ : Comme la fonction $t\mapsto\frac{1}{t^{\alpha}}$ est strictement décroissante sur $]0,+\infty[$, on obtient la même double inégalité que ci-dessus que dans le cas où $0<\alpha<1$, donc pour tout $n\geqslant2$, on a donc :
    $$\ds\frac{(n+1)^{1-\alpha}}{1-\alpha}-\frac{1}{1-\alpha}\leqslant\sum_{k=1}^{n}{\frac{1}{k^{\alpha}}}\leqslant\frac{n^{1-\alpha}}{1-\alpha}-\frac{\alpha}{1-\alpha}$$$$\ds\frac{1}{\alpha-1}-\frac{1}{(\alpha-1)(n+1)^{\alpha-1}}\leqslant\sum_{k=1}^{n}{\frac{1}{k^{\alpha}}}\leqslant\frac{\alpha}{\alpha-1}-\frac{1}{(\alpha-1)n^{\alpha-1}}\leqslant\frac{\alpha}{\alpha-1}$$Comme la suite des sommes partielles est croissante (termes positifs) et majorée alors la série converge.\\ Attention toutefois, le majorant n'est pas égal à la somme de la série, on peut seulement écrire que :\\ $$\ds\frac{1}{\alpha-1}\leqslant\sum_{k=1}^{+\infty}{\frac{1}{k^{\alpha}}}\leqslant\frac{\alpha}{\alpha-1}=1+\frac{1}{\alpha-1}$$
math/2/demo/serie_riemann.txt · Dernière modification : 2020/05/10 21:31 de 127.0.0.1